Đến nội dung

Hình ảnh

Topic tổng hợp một số bất đẳng thức trong kì thi MO các nước

* * * * * 16 Bình chọn

  • Please log in to reply
Chủ đề này có 501 trả lời

#481
khanghaxuan

khanghaxuan

    Trung úy

  • Thành viên
  • 969 Bài viết

Giải bài 173 : 

Đặt : $\left\{\begin{matrix} a+2b+c=x & & \\ a+b+2c=y & & \\ a+b+3c=z & & \end{matrix}\right.\Rightarrow \left\{\begin{matrix} a+3c=2y-x & & \\ b=x+z-2y & & \\ c=z-y & & \end{matrix}\right.$

do đó ta có : $f(x,y,z)=2(\frac{y}{x}+\frac{2x}{y})+4(\frac{z}{y}+\frac{2y}{z})-17$

Tới đây thì đánh giá bằng CS rồi tìm dấu = là xong :))


Bài viết đã được chỉnh sửa nội dung bởi khanghaxuan: 08-08-2015 - 15:37

Điều tôi muốn biết trước tiên không phải là bạn đã thất bại ra sao mà là bạn đã chấp nhận nó như thế nào .

- A.Lincoln -

#482
Hoang Nhat Tuan

Hoang Nhat Tuan

    Hỏa Long

  • Thành viên
  • 974 Bài viết

Vực topic dậy nào  :icon6:

Bài 173:(China Girls Math Olympiad)

Cho $a,b,c$ là các số thực dương.Tìm Min của:

$ \frac {a + 3c}{a + 2b + c} + \frac {4b}{a + b + 2c} - \frac {8c}{a + b + 3c}.$

Bài 174:(China Girls Math Olympiad)

Cho $u;v;w$ là các số thực dương thoả mãn $ u\sqrt {vw} + v\sqrt {wu}+ w\sqrt {uv} \geq 1$.Tìm Min của $ u + v+ w$

Bài 173: Đặt: $x=a+2b+c;y=a+b+2c;z=a+b+3c$

Cần tìm Min của:

$\frac{2y-x}{x}+\frac{4(z+x-2y)}{y}-\frac{8(z-y)}{z}=-17+2(\frac{y}{x}+\frac{2x}{y})+4(\frac{2y}{z}+\frac{z}{y})$

$\geq 4\sqrt{2}+8\sqrt{2}-17=12\sqrt{2}-17$


Ngài có thể trói cơ thể tôi, buộc tay tôi, điều khiển hành động của tôi: ngài mạnh nhất, và xã hội cho ngài thêm quyền lực; nhưng với ý chí của tôi, thưa ngài, ngài không thể làm gì được.

#483
hoctrocuaZel

hoctrocuaZel

    Thượng úy

  • Thành viên
  • 1162 Bài viết

Chào bạn :)) , bạn nói rõ hơn phương pháp này được không :) Tại sao lại đặt được như thế và nếu có tài liệu về phương pháp này thì càng tốt nhé :))

Phương pháp trội tử tức là giá trị bằng $1/2$ cộng thêm $1$ ẩn để thêm vào nữa.

Bài tập:

$(1)$

$\sum (\frac{a}{a+b})^2+3\geqslant \frac{5}{2}\sum \frac{a}{a+b}$

Vân vân :v


Hướng TH Phan
$(1)$ Lòng như mây trắng
$(2)$: Forever Young
$(3)$: You are the apple of my eye
Người ta thường nói tuổi thanh xuân như một cơn mưa rào, nếu bị ướt một lần thì bạn vẫn mong muốn thêm 1 lần nữa ...
#hoctrocuaZel
:(

#484
hoctrocuaHolmes

hoctrocuaHolmes

    Thượng úy

  • Thành viên
  • 1013 Bài viết

Bài 175: (China Girls Math Olympiad) Cho $a_{1};a_{2};...a_{n}$ là các số không âm.Chứng minh rằng $\frac{1}{1+ a_1}+\frac{ a_1}{(1+ a_1)(1+ a_2)}+\frac{ a_1 a_2}{(1+ a_1)(1+ a_2)(1+ a_3)}+...+\frac{ a_1 a_2... a_{n-1}}{(1+ a_1)(1+ a_2)... (1+ a_n)} \leq 1.$

Bài 176: (Romania) Cho $n\epsilon N*$ và $a_{1};a_{2};...a_{n}\epsilon R$ thoả mãn ${a}_{1}}+{{a}_{2}}+...+{{a}_{k}\leq k$ với $\forall k\epsilon (1;2;..;n)$.Chứng minh rằng $\frac{{{a}_{1}}}{1}+\frac{{{a}_{2}}}{2}+...+\frac{{{a}_{n}}}{n}\leq \frac{1}{1}+\frac{1}{2}+...+\frac{1}{n}$


Bài viết đã được chỉnh sửa nội dung bởi votruc: 11-08-2015 - 08:57


#485
Master Yi

Master Yi

    Lính mới

  • Thành viên mới
  • 2 Bài viết

\[\frac{a}{\sqrt{a+b}}+\frac{b}{\sqrt{b+c}}+\frac{c}{\sqrt{c+a}}\leq 4\sqrt{a+b+c}\]
 

Với mọi a,b,c ko âm,CM bđt trên đúng


Bài viết đã được chỉnh sửa nội dung bởi Master Yi: 03-05-2016 - 01:22


#486
tanthanh112001

tanthanh112001

    Sĩ quan

  • Thành viên
  • 315 Bài viết

Bài 177: (USAMO 2003) Cho 3 số thực dương $a,b,c$. Chứng minh rằng:

$\frac{(2a+b+c)^2}{2a^2+(b+c)^2}+\frac{(2b+c+a)^2}{2b^2+(c+a)^2}+\frac{(2c+a+b)^2}{2c^2+(a+b)^2}\leq 8$.


:ukliam2: TINH HOA CỦA TOÁN HỌC LÀ NẰM Ở SỰ TỰ DO CỦA NÓ. :ukliam2: 

---- Georg Cantor ----

 

996a71363a3740db895ba753827984fd.1.gif


#487
tpdtthltvp

tpdtthltvp

    Trung úy

  • Điều hành viên THCS
  • 831 Bài viết

Bài 177: (USAMO 2003) Cho 3 số thực dương $a,b,c$. Chứng minh rằng:

$\frac{(2a+b+c)^2}{2a^2+(b+c)^2}+\frac{(2b+c+a)^2}{2b^2+(c+a)^2}+\frac{(2c+a+b)^2}{2c^2+(a+b)^2}\leq 8$.

Chuẩn hóa $a+b+c=3$. Bất đẳng thức cần chứng minh tương đương với:

$$\sum \frac{(a+3)^2}{2a^2+(3-a)^2}\leq 8\Leftrightarrow \sum \frac{a^2+6a+9}{3a^2-6a+9}\leq 8$$

Mặt khác, ta có:

$$\frac{a^2+6a+9}{3a^2-6a+9}\leq \frac{4a}{3}+\frac{4}{3}\Leftrightarrow \frac{(a-1)^2(4a+3)}{3(a^2-2a+3)}\geq 0(\text{luôn đúng})$$

Tương tự, ta có:

$$\frac{b^2+6b+9}{3b^2-6b+9}\leq \frac{4b}{3}+\frac{4}{3}$$

$$\frac{c^2+6c+9}{3c^2-6c+9}\leq \frac{4c}{3}+\frac{4}{3}$$

Cộng vế với vế $3$ bất đẳng thức trên ta có đpcm.


Bài viết đã được chỉnh sửa nội dung bởi tpdtthltvp: 22-05-2016 - 08:17

$\color{red}{\mathrm{\text{How I wish I could recollect, of circle roud}}}$

$\color{red}{\mathrm{\text{The exact relation Archimede unwound ! }}}$

 


#488
cristianoronaldo

cristianoronaldo

    Thượng sĩ

  • Thành viên
  • 233 Bài viết

Bài 177: (USAMO 2003) Cho 3 số thực dương $a,b,c$. Chứng minh rằng:

P=$\frac{(2a+b+c)^2}{2a^2+(b+c)^2}+\frac{(2b+c+a)^2}{2b^2+(c+a)^2}+\frac{(2c+a+b)^2}{2c^2+(a+b)^2}\leq 8$.

Second Solution:

Bất đẳng thức cần chứng minh tương đương với:

$\sum \frac{(2+\frac{b+c}{a})^2}{2+(\frac{b+c}{a})^2}\leq 8$

Đặt$\left\{\begin{matrix} \frac{b+c}{a}=x\\ \frac{c+a}{b}=y \\ \frac{a+b}{c}=z\end{matrix}\right.$.Khi đó ta có:xyz=x+y+z+2

Bât đẳng thức cần chứng minh tương đương với:

P=$\sum \frac{(2+x)^2}{2+x^2}\leq 8\Leftrightarrow \sum \frac{(x-1)^2}{x^2+2}\geq \frac{1}{2}$

Áp dụng bất đẳng thức Cauchy-Schwarz ta có:

P$P\geq \frac{(x+y+z-3)^2}{x^2+y^2+z^2+g}$

Cuối cùng ta cần chứng minh:

$2(x+y+z-3)^2\geq x^2+y^2+z^2+6\Leftrightarrow 2(x+y+z-3)^2\geq (x+y+z)^2-2(xy+yz+zx)+6$
Mặt khác dễ dàng chỉ ra được $xyz\geq 8$.Từ đó ta được;$xy+yz+zx\geq 12,x+y+z\geq 6$

Đặt x+y+z=t thì bất đẳng thức cần chứng minh trở thành:

$2(t-3)^2\geq t^2-18\Leftrightarrow (t-3)(t-6)\geq 0$(luôn đúng)

Dấu ''='' xảy ra khi a=b=c


Nothing in your eyes


#489
yeutoanmanhliet

yeutoanmanhliet

    Binh nhất

  • Thành viên mới
  • 35 Bài viết

Câu 7 : $\sum \frac{x^{3}}{z^{3}+x^{2}y}=\sum \frac{x^{4}}{xz^{3}+x^{3}y}\geq \frac{(\sum x^{2})^{2}}{\sum xy(x^{2}+y^{2})}$

Mặt khác , ta có : $(\sum x^{2})^{2}\geq \sum x^{3}y$

Nên ta có : $VT\geq \frac{3}{2}$

cái thứ 2 chứng minh sao anh



#490
Minato Namikaze

Minato Namikaze

    Lính mới

  • Thành viên mới
  • 7 Bài viết
Bài 2 khá dễ ta đặt √a=x;√b=y;√c=z khi đó thoả x^2y^2+y^2z^2+x^2z^2=1 từ đó ta cm đc 3√3×x^2y^2z^2=<1 và điều cần cm là x^3/(y^2z^2)+y^3/(x^2z^2)+z^3/(x^2y^2) >=√3×(x+y+z)

#491
Minato Namikaze

Minato Namikaze

    Lính mới

  • Thành viên mới
  • 7 Bài viết
Cauchy schwarz ta đc điều cần cm là x+y+z>=3√3×xyz điều này tuong duong voi 3√3×x^2×y^2×z^2 =<1

#492
9nho10mong

9nho10mong

    Binh nhất

  • Thành viên
  • 38 Bài viết

Bài 177: (USAMO 2003) Cho 3 số thực dương $a,b,c$. Chứng minh rằng:

$\frac{(2a+b+c)^2}{2a^2+(b+c)^2}+\frac{(2b+c+a)^2}{2b^2+(c+a)^2}+\frac{(2c+a+b)^2}{2c^2+(a+b)^2}\leq 8$.

 

Với $a,b,c > 0 $ có

$$ \dfrac{8}{3} + \dfrac{4 \left( 2a-b-c \right)}{3 \left( a+b+c \right)} - \dfrac{\left( 2a+b+c \right)^2}{2a^2 + \left( b+c \right)^2} = \dfrac{\left( 5a+b+c \right) \left( 2a-b-c \right)^2}{3 \left( a+b+c \right) \left( 2a^2 + \left( b+c \right)^2\right)} \ge 0 $$

Từ đó có

$$ \dfrac{\left( 2a+b+c \right)^2}{2a^2 + \left( b+c \right)^2} \le  \dfrac{8}{3} + \dfrac{4 \left( 2a-b-c \right)}{3 \left( a+b+c \right)}$$

Suy ra

$$  \sum  \dfrac{\left( 2a+b+c \right)^2}{2a^2 + \left( b+c \right)^2} \le 8$$

Đó là điều cần chứng minh.


Bài viết đã được chỉnh sửa nội dung bởi 9nho10mong: 11-02-2017 - 11:40

.

 


#493
Nguyenhuyen_AG

Nguyenhuyen_AG

    Trung úy

  • Thành viên nổi bật 2016
  • 945 Bài viết

Cho ba số thực $a,b,c$ thỏa mãn $a^2+b^2+c^2>0.$ Chứng minh rằng:

$$\frac{(2a+b+c)^2}{2a^2+(b+c)^2}+\frac{(2b+c+a)^2}{2b^2+(c+a)^2}+\frac{(2c+a+b)^2}{2c^2+(a+b)^2}\leq 8.$$


Nguyen Van Huyen
Ho Chi Minh City University Of Transport

#494
Nguyenhuyen_AG

Nguyenhuyen_AG

    Trung úy

  • Thành viên nổi bật 2016
  • 945 Bài viết

\[\frac{a}{\sqrt{a+b}}+\frac{b}{\sqrt{b+c}}+\frac{c}{\sqrt{c+a}}\leq 4\sqrt{a+b+c}\]
 

Với mọi a,b,c ko âm,CM bđt trên đúng

 

Ta chứng minh bất đẳng thức chặt hơn sau đây

\[\frac{a}{\sqrt{a+b}}+\frac{b}{\sqrt{b+c}}+\frac{c}{\sqrt{c+a}}\leq \frac54\sqrt{a+b+c}.\]

Lời giải.


Nguyen Van Huyen
Ho Chi Minh City University Of Transport

#495
Nguyenhuyen_AG

Nguyenhuyen_AG

    Trung úy

  • Thành viên nổi bật 2016
  • 945 Bài viết

Phương pháp trội tử tức là giá trị bằng $1/2$ cộng thêm $1$ ẩn để thêm vào nữa.

Bài tập:

$(1)$

$\sum (\frac{a}{a+b})^2+3\geqslant \frac{5}{2}\sum \frac{a}{a+b}$

Vân vân :v

 

Ta có

\[\sum \frac{a^2}{(a+b)^2} +3 - \frac{5}{2}\sum \frac{a}{a+b} = \frac{1}{2(a+b)^2(b+c)^2(c+a)^2}\sum ca(ab+3bc+3ca+c^2)(a-b)^2 \geqslant 0.\]


Nguyen Van Huyen
Ho Chi Minh City University Of Transport

#496
viet9a14124869

viet9a14124869

    Trung úy

  • Thành viên
  • 903 Bài viết

hình như có người gửi rồi hay sao ý ^-^

Bài 168(Hungary MO): Cho a,b,c>0. CMR nếu a+b+c= 2(ab+ca+ab). Tìm Min biểu thức sau:

P=$\sum \frac{a^3+b^2}{a^2+b^2}$

Ta chứng minh một bổ đề sau

$\frac{a^3+b^3}{a^2+b^2}\geq \frac{a+b}{2}\Leftrightarrow 2(a^3+b^3)\geq (a^2+b^2)(a+b)\Leftrightarrow (a-b)^2(a+b)\geq 0$ (luôn đúng)

Mặt khác $a+b+c=2ab+2bc+2ca\leq \frac{2}{3}(a+b+c)^2\Rightarrow a+b+c\geq \frac{3}{2}$

Do đó $\sum \frac{a^3+b^3}{a^2+b^2}\geq \sum \frac{a+b}{2}=a+b+c\geq \frac{3}{2}\Leftrightarrow a=b=c=\frac{1}{2}$


Bài viết đã được chỉnh sửa nội dung bởi viet9a14124869: 11-02-2017 - 19:05

                                                                    SÓNG BẮT ĐẦU TỪ GIÓ

                                                                    GIÓ BẮT ĐẦU TỪ ĐÂU ?

                                                                    ANH CŨNG KHÔNG BIẾT NỮA 

                                                                    KHI NÀO...? TA YÊU NHAU .


#497
Minhnksc

Minhnksc

    Sĩ quan

  • Điều hành viên OLYMPIC
  • 302 Bài viết

hình như có người gửi rồi hay sao ý ^-^

Ta chứng minh một bổ đề sau

$\frac{a^3+b^3}{a^2+b^2}\geq \frac{a+b}{2}\Leftrightarrow 2(a^3+b^3)\geq (a^2+b^2)(a+b)\Leftrightarrow (a-b)^2(a+b)\geq 0$ (luôn đúng)

Mặt khác $a+b+c=2ab+2bc+2ca\leq \frac{2}{3}(a+b+c)^2\Rightarrow a+b+c\geq \frac{3}{2}$

Do đó $\sum \frac{a^3+b^3}{a^2+b^2}\geq \sum \frac{a+b}{2}=a+b+c\geq \frac{3}{2}\Leftrightarrow a=b=c=\frac{1}{2}$

bạn đọc sai đề bài rồi, VT của đề là $\sum\frac{a^3+b^2}{a^2+b^2}$ chứ không phải là $\sum\frac{a^3+b^3}{a^2+b^2}$


Sống khỏe và sống tốt :D


#498
AnhTran2911

AnhTran2911

    Thượng sĩ

  • Thành viên
  • 230 Bài viết

BÀI TOÁN (Serbian National Olympiad 2008):

Cho các số thực dương x,y,z sao cho $x+y+z=1$. 

CMR: $\sum\frac{1}{yz+x+\frac{1}{x}}\leq\frac{27}{31}$

 


        AQ02

                                 


#499
Drago

Drago

    Sĩ quan

  • Thành viên
  • 462 Bài viết

BÀI TOÁN (Serbian National Olympiad 2008):

Cho các số thực dương x,y,z sao cho $x+y+z=1$. 

CMR: $\sum\frac{1}{yz+x+\frac{1}{x}}\leq\frac{27}{31}$

18718237_1858025814446137_1967009653_n.p


$\mathbb{VTL}$


#500
Uchiha sisui

Uchiha sisui

    Trung sĩ

  • Thành viên
  • 196 Bài viết

Xin up lại bài toán hay sau của anh Dragon, bài này thấy anh up mà chưa có giải nên em xin up lại vào topic luôn :)

 

Bài toán. Cho a, b, c là các số thực dương thỏa mãn $ab+bc+ca=3abc$. Chứng minh rằng: 

 

$\sum \frac{1}{\sqrt[3]{a^{3}+bc}}\leq \frac{3}{2}$






1 người đang xem chủ đề

0 thành viên, 1 khách, 0 thành viên ẩn danh